LSAT and Law School Admissions Forum

Get expert LSAT preparation and law school admissions advice from PowerScore Test Preparation.

 Rachael Wilkenfeld
PowerScore Staff
  • PowerScore Staff
  • Posts: 1358
  • Joined: Dec 15, 2011
|
#96546
Hi burneagle,

Answer choice (A) is a mistaken negation. We can't conclude anything about a government being unjustified in interfering. When we look at an answer choice like this one, we want to think about which part of the answer choice is sufficient, and which part is necessary. Typically, that will look like it does here. The premises are suggested as the sufficient conditions, while the conclusion is what the author draws as necessary.

Our original conditional is as follows:

increase likelihood of physical harm to others AND not motivated by desire to help others :arrow: government justified in interfering.

Answer choice (A) describes a situation where there is no harm to anyone, and the individual is motivated by a desire to help others. It then goes to conclude that the government would not be justified in interfering.

This would be drawn out like so:

likelihood of physical harm to others and motivated by a desire to help others :arrow: government justified in interfering

It doesn't particularly matter if both parts of the sufficient condition are negated---it's the fact that it's a mistaken negation that's a problem, not that both sufficient conditions were negated.

Hope that helps!

Get the most out of your LSAT Prep Plus subscription.

Analyze and track your performance with our Testing and Analytics Package.